Un número épsilon es un ordinal $\epsilon$ tal que $\epsilon=\omega^\epsilon.$ ¿Cuál es la cardinalidad del conjunto de todos los números épsilon menores que $\omega_1$ ?
Lo pregunto por una prueba que acabo de leer y que parece presuponer que hay un número contable de tales ordinales, y me parece intuitivamente que debería haber un número incontable (aunque no sé cómo demostrarlo).
Añadido. Vale, acabo de entender que la prueba que he mencionado está bien aunque haya incontables ordinales de este tipo, pero sigo sin ver cómo puedo encontrar su número.
3 votos
$\omega_1$ . ${}$
0 votos
@Andres Gracias, eso es lo que pensé. Pero, ¿cómo puedo probarlo?
1 votos
OK, finalmente lo encontré. La razón por la que no me gusta la respuesta estándar (como las de abajo) es que utiliza la elección (en la forma: La unión contable de conjuntos contables es contable), pero el hecho de que haya $\omega_1$ -muchos $\varepsilon$ -Números abajo $\omega_1$ no necesita elección. Esto se preguntó en este sitio antes, aquí está el enlace: math.stackexchange.com/q/181424/462
0 votos
Ahora bien, esto es sobre todo una manía mía, pero creo que es útil ser consciente de estas cosas. Permítanme ir más allá: Hay un truco estándar para mostrar que algunos resultados que utilizan la elección no la requieren. Es metamatemático, nos movemos entre el universo y un modelo interno de elección, y comprobamos que cualquier hecho relevante verificado en el modelo interno se mantiene en el universo. Esta es la forma de la respuesta de Joel en el enlace anterior, por lo que prefiero la de Tomasz, ya que es puramente combinatoria. Para otro ejemplo, véase aquí: mathoverflow.net/questions/40507/
0 votos
Andrés, el problema en este caso de subir y bajar entre $L$ y $V$ es que el único momento en que necesitamos esto es cuando $\omega_1$ es singular, en cuyo caso $\omega_1^L\neq\omega_1$ por lo que la prueba requiere algún aumento. Creo, quien quiera que sea, que probablemente se pueda demostrar un lema interesante de que si los ordinales son una secuencia iterativa como la de mi respuesta, entonces podemos enumerarlos uniformemente, en cuyo caso no se necesita ninguna elección para demostrar que su límite es contable.
1 votos
@Andres: Ah, en realidad no hace falta ningún truco. La aritmética ordinal es absoluta, así que aunque $\omega_1^L\neq\omega_1$ , en $L$ existe una biyección entre el $\varepsilon$ números que son menores que $\omega_1^V$ y $\omega_1^V$ que atestiguarían que hay $\aleph_1$ contable $\varepsilon$ números en $V$ .